Mathcenter Forum

Mathcenter Forum (https://www.mathcenter.net/forum/index.php)
-   อสมการ (https://www.mathcenter.net/forum/forumdisplay.php?f=18)
-   -   Nice inequality problem (https://www.mathcenter.net/forum/showthread.php?t=6197)

RoSe-JoKer 30 ธันวาคม 2008 22:12

Nice inequality problem
 
Prove that
for all $a,b,c\geq 0$
$3(a+b+c) \geq 2(\sqrt{a^2+bc}+\sqrt{b^2+ca}+\sqrt{c^2+ab})$
:great:

warutT 02 มกราคม 2009 14:04

อ้างอิง:

ข้อความเดิมเขียนโดยคุณ RoSe-JoKer (ข้อความที่ 46780)
Prove that
$3(a+b+c) \geq 2(\sqrt{a^2+bc}+\sqrt{b^2+ca}+\sqrt{c^2+ca})$
:great:

เปลี่ยนเป็น $ab$ หรือเปล่าครับ
ผมคิดว่า $a,b,c \geq 0$ ด้วยนะครับ
ไม่รู้ถูกหรือเปล่าช่วยเช็คให้ด้วยครับ :please:

Mathematica 02 มกราคม 2009 15:35

สวยดีนะครับข้อนี้ คิดว่าวิธีข้างบนน่าจะถูกแล้วนะครับ
แต่ว่าอสมการจะ hold เมื่อไรครับ ???
ผมคิดว่า hold แต่ $a,b,c=0$ ครับ

nooonuii 03 มกราคม 2009 02:05

อ้างอิง:

ข้อความเดิมเขียนโดยคุณ Mathematica (ข้อความที่ 46963)
สวยดีนะครับข้อนี้ คิดว่าวิธีข้างบนน่าจะถูกแล้วนะครับ
แต่ว่าอสมการจะ hold เมื่อไรครับ ???
ผมคิดว่า hold แต่ $a,b,c=0$ ครับ

สมการเป็นจริงเมื่อตัวใดตัวหนึ่งเป็นศูนย์ และอีกสองตัวเท่ากันครับ

RoSe-JoKer 03 มกราคม 2009 19:47

อ้างอิง:

ข้อความเดิมเขียนโดยคุณ warutT (ข้อความที่ 46958)
เปลี่ยนเป็น $ab$ หรือเปล่าครับ
ผมคิดว่า $a,b,c \geq 0$ ด้วยนะครับ
ไม่รู้ถูกหรือเปล่าช่วยเช็คให้ด้วยครับ :please:

ขอโทษด้วยนะครับที่พิมพ์โจทย์ผิด...ต้องเปลี่ยนเป็น ab นะครับ (แต่จริงๆผมน่าจะรู้ๆกันอยู่แล้วว่าผมพิมพ์ผิด)
ว่าแต่คุณ Mathematica ได้อ่านวิธีทำดูดีๆหรือยังครับเนี่ย?? คือว่าที่คุณ WarutT ทำผิดก็คงเนื่องมาจากอ่านโจทย์ผิดหล่ะมั้งครับ ดูโจทย์ใหม่ดีๆนะครับ $2\sum_{cyc} \sqrt{a^2+bc}$

warutT 03 มกราคม 2009 20:46

อ้างอิง:

ข้อความเดิมเขียนโดยคุณ RoSe-JoKer (ข้อความที่ 47042)
ขอโทษด้วยนะครับที่พิมพ์โจทย์ผิด...ต้องเปลี่ยนเป็น ab นะครับ (แต่จริงๆผมน่าจะรู้ๆกันอยู่แล้วว่าผมพิมพ์ผิด)
ว่าแต่คุณ Mathematica ได้อ่านวิธีทำดูดีๆหรือยังครับเนี่ย?? คือว่าที่คุณ WarutT ทำผิดก็คงเนื่องมาจากอ่านโจทย์ผิดหล่ะมั้งครับ ดูโจทย์ใหม่ดีๆนะครับ $2\sum_{cyc} \sqrt{a^2+bc}$

ขอโทษครับ :please: ลืม :p
ผมจะลองทำอีกครั้งนึงครับ

warutT 03 มกราคม 2009 21:19

ถ้า $a,b,c \geq 0$ นะครับ
$-(ab+bc+ca) \leqslant 0 ...(1)$
From Cauchy-Schwarz Inequality
$a\sqrt{a-b}\sqrt{a-b}+b\sqrt{b-c}\sqrt{b-c}+c\sqrt{c-a}\sqrt{c-a}$
$\leqslant \sqrt{a^2(a-b)+b^2(b-c)+c^2(c-a)}\sqrt{a-b+b-c+c-a}=0$
$a(a-b)+b(b-c)+c(c-a) \leqslant 0 ...(2)$
$(1)+(2);a(a-b)+b(b-c)+c(c-a)-(ab+bc+ca) \leqslant 0$
$a^2+b^2+c^2-2(ab+bc+ca) \leqslant 0$
$3(a^2+b^2+c^2)-6(ab+bc+ca) \leqslant 0$
$12(a^2+b^2+c^2+ab+bc+ca)-9(a^2+b^2+c^2+2ab+2bc+2ca) \leqslant 0$
$12(a^2+b^2+c^2+ab+bc+ca) \leqslant 9(a^2+b^2+c^2+2ab+2bc+2ca)$
$3(a^2+b^2+c^2+ab+bc+ca) \leqslant \frac{9}{4}(a+b+c)^2 ...(3)$
From Cauchy-Schwarz Inequality
$2(\sqrt{a^2+bc}+\sqrt{b^2+ca}+\sqrt{c^2+ab}) \leq 2(\sqrt{3}\sqrt{a^2+b^2+c^2+ab+bc+ca})$
$=2(\sqrt{3(a^2+b^2+c^2+ab+bc+ca)})$
$\leq 2(\sqrt{\frac{9}{4}(a+b+c)^2}) (from(3))$
$=3(a+b+c)$
$\therefore 2(\sqrt{a^2+bc}+\sqrt{b^2+ca}+\sqrt{c^2+ab}) \leq 3(a+b+c)$ :great:

beginner01 03 มกราคม 2009 21:25

อ้างอิง:

ข้อความเดิมเขียนโดยคุณ warutT (ข้อความที่ 47060)
...
From Cauchy-Schwarz Inequality
$a\sqrt{a-b}\sqrt{a-b}+b\sqrt{b-c}\sqrt{b-c}+c\sqrt{c-a}\sqrt{c-a}$
$\leqslant \sqrt{a^2(a-b)+b^2(b-c)+c^2(c-a)}\sqrt{a-b+b-c+c-a}=0$
...

สามารถแยกแบบนี้ได้เหรอครับ :eek:
การแยกที่มีรูทติดอยู่จะต้องมั่นใจว่าภายใต้รูทนั้น จะต้องมีค่าไม่น้อยกว่า 0 ซึ่งการแยกแบบนี้จะได้ว่า $a\geq b$, $b\geq c$, $c\geq a$ ซึ่งจะไปบีบบังคับให้ $a=b=c$ นะครับ
แถมอีกนิดนึง: $a^2+b^2+c^2\geq ab+bc+ca$ ครับ
ว่าแต่ขอถาม จขกท. ว่า a,b,c เป็นจำนวนจริงบวกหรือเปล่าครับ?

Mathematica 03 มกราคม 2009 22:22

จริงด้วยครับคุณโรสโจ๊กเกอร์
โจทย์เป็น จริงบวกรวม 0 อยู่แล้วครับ :great:

wttskt 05 มกราคม 2009 11:50

สมมติว่า $a\geq b\geq c$
กรณีที่ b=c=0 เห็นได้ชัดว่าอสมการเป็นจริง
สมมติ $b > 0$

หารอสมการทั้งสองข้างด้วย a และแทน x^2=b/a และ y=c/a จะได้ว่า
$ 3(1+x^2+y) \geq 2(\sqrt{1+x^2 y}+\sqrt{x^4+y}+\sqrt{x^2+y^2}) $
โดยที่ $1\geq x^2 \geq y$

เนื่องจาก $\sqrt{1+x} \leq 1+x/2$ สำหรับทุกจำนวนจริง x
ดังนั้น:
$\sqrt{1+x^2 y} \leq 1+ x^2 y/2$
$\sqrt{x^4+y} = x^2 \sqrt{1+y/x^4} \leq x^2(1+\frac{y}{2x^4})=x^2+\frac{y}{2x^2}$
$\sqrt{x^2+y^2} = ... \leq x(1+\frac{y^2}{2x^2})=x+\frac{y^2}{2x}$

=> $2(\sqrt{1+x^2 y}+\sqrt{x^4+y}+\sqrt{x^2+y^2}) \leq 2(1+ \frac{x^2 y}{2} +x^2+\frac{y}{2x^2}+x+\frac{y^2}{2x})$

และจาก $1\geq x^2\geq y\geq 0$ ได้ว่า
$1-y \geq x –y$ , $1+x \geq 1+\sqrt{y} \geq 2y$ => $1-y \geq y-x$
และ $2x \geq 2x-y$, $2x \geq 2\sqrt{y} \geq y-2x$
ทำให้ได้ว่า
$(1-y)(x^4+1) \geq (1-y)(2x^2) = x(1-y)(2x)\geq x| x-y | | 2x-y | \geq x(x-y)(2x-y)$
=> $(1-y)x^4+3x^2y+(1-y) \geq 2x^3+xy^2$
=> $3(1+x^2+y) \geq 2(1+ \frac{x^2 y}{2} +x^2+\frac{y}{2x^2}+x+\frac{y^2}{2x})
\geq 2(\sqrt{1+x^2 y}+\sqrt{x^4+y}+\sqrt{x^2+y^2}) $.

RoSe-JoKer 05 มกราคม 2009 21:28

อ้างอิง:

ข้อความเดิมเขียนโดยคุณ wttskt (ข้อความที่ 47217)
...
ทำให้ได้ว่า
$(1-y)(x^4+1) \geq (1-y)(2x^2) = x(1-y)(2x)\geq x| x-y | | 2x-y | \geq x(x-y)(2x-y)$
=> $(1-y)x^4+3x^2y+(1-y) \geq 2x^3+xy^2$
=> $3(1+x^2+y) \geq 2(1+ \frac{x^2 y}{2} +x^2+\frac{y}{2x^2}+x+\frac{y^2}{2x})
\geq 2(\sqrt{1+x^2 y}+\sqrt{x^4+y}+\sqrt{x^2+y^2}) $.
...

$3(1+x^2+y) \geq 2(1+ \frac{x^2 y}{2} +x^2+\frac{y}{2x^2}+x+\frac{y^2}{2x})$
อธิบายตรงนี้หน่อยได้ไหมครับ จะขอบคุณมากๆเลยครับ เพราะที่ผมลองทำต่อจากตรง
$(1-y)x^4+3x^2y+(1-y) \geq 2x^3+xy^2$
ของคุณ wttskt แล้วผมรู้สึกว่าอสมการที่ผมได้จะกลับข้างนิดหน่อยนะครับ
ขอบคุณมากๆครับ สำหรับไอเดียใหม่ๆดีๆ :)

wttskt 05 มกราคม 2009 22:24

:mellow: :mellow: ดูเหมือนว่าผมจะกระจายบางพจน์ผิดไปหน่อยนะครับ


เวลาที่แสดงทั้งหมด เป็นเวลาที่ประเทศไทย (GMT +7) ขณะนี้เป็นเวลา 22:34

Powered by vBulletin® Copyright ©2000 - 2024, Jelsoft Enterprises Ltd.
Modified by Jetsada Karnpracha